¿Qué sucede con el Lagrangiano de la teoría de Dirac bajo conjugación de carga?

Considere un operador de conjugación de carga que actúa sobre el campo de Dirac ( ψ ) como

ψ C C ψ C 1 = C γ 0 T ψ
Así como podemos operar el operador de paridad en el Lagrangiano, y decimos que una teoría tiene una simetría si
PAG L ( t , X i ) PAG 1 = L ( t , X i )

Supongamos que operamos C en el Lagrangiano de Dirac, ¿qué debemos obtener?

C L D i r a C ( X m ) C 1 = L D i r a C ( X m ) ?
en analogía a la transformación del campo escalar ϕ bajo conjugación de carga.

En la misma nota uno puede preguntar qué ecuación debería ψ C ¿satisfacer? En caso de que satisfaga la ecuación de Dirac conjugada como

( i γ m m + metro ) ψ C = 0 ?
Si es así, ¿alguien puede darme la interpretación física para ello?

Estoy haciendo esta pregunta porque quiero usar explícitamente ψ C y comprobar si mantiene invariante el Lagrangiano de Dirac. He hecho un cálculo sustituyendo ψ C en la ecuación de Dirac y he encontrado que no es satisfactoria como se muestra a continuación.

( i γ m m metro ) ψ C = i ( γ m C γ 0 T ) m ψ ( C γ 0 T ) metro ψ
Usaremos C 1 γ m C = γ m T y { γ m T , γ v T } = 2 gramo m v .
Considerar
γ m C γ 0 T = C C 1 γ m C γ 0 T = C γ m T γ 0 T = C γ 0 T γ m T
Por lo tanto, sustituyendo de nuevo obtendremos
( i γ m m metro ) ψ C = C γ 0 T ( i γ m T m ψ metro ψ ) = C γ 0 T [ ( i γ m T m ψ metro ψ ) T ] T = C γ 0 T ( i ψ γ m m metro ψ ) T = C γ 0 T [ ( i ψ ¯ γ 0 γ m m metro ψ ¯ γ 0 ) ] T
Ahora
γ 0 γ m m = ( γ 0 t + γ i i ) γ 0
Si volvemos a sustituir obtendremos
( i γ m m metro ) ψ C = C γ 0 T [ { i ψ ¯ ( γ 0 t + γ i i ) metro ψ ¯ } γ 0 ] T 0

Respuestas (2)

La respuesta corta a la pregunta "¿Qué le sucede al Lagrangiano de la teoría de Dirac bajo la conjugación de carga?" no es nada." Es invariante con respecto a la conjugación de carga.

Antes de llegar a la exposición más larga, me gustaría señalar un malentendido potencial sobre la naturaleza de la invariancia de las ecuaciones de movimiento bajo transformaciones de simetría que surge sobre su declaración sobre la paridad ( X X ). tu ecuación

PAG L ( t , X i ) PAG 1 = L ( t , X i )
es correcto. Pero esto, en sí mismo, no "dice que una teoría tenga simetría". De hecho, impone una restricción a la teoría: que la teoría debe ser una función par del vector de posición. Por ejemplo, para un campo escalar real, ϕ ( t , X ) , el operador de energía cinética en la densidad lagrangiana, m m ϕ ( X ) es invariante bajo paridad. De hecho, es sólo la acción ( S = d 4 X L ) que necesitan ser invariantes bajo la transformación de simetría. (A menudo, esto se reduce a la invariancia de la densidad lagrangiana). Por lo tanto, no es necesario que cada término de la (densidad) lagrangiana sea invariante (aunque este suele ser el caso).

Volviendo a la ecuación de Dirac, la declaración completa, en palabras, de la invariancia de la teoría de la interacción de los electrones con la luz (QED) bajo las transformaciones discretas ( PAG , C , & T ) es que la teoría es invariante bajo cada uno de ellos por separado o en cualquier combinación. (QED es menos "interesante" que la teoría electrodébil en este sentido, ya que la teoría electrodébil parece violar las tres por separado, pero quizás no todas simultáneamente).

Debemos recordar que la invariancia bajo C requiere la transformación no sólo de la función de onda, ψ C C ψ C 1 = C γ 0 T ψ sino también el cargo, q q . En el caso de la ecuación libre de Dirac/Lagrangiana considerada anteriormente, la carga no aparece, por lo que no es directamente relevante para la presente discusión, pero es importante tenerla en cuenta.

Ahora las respuestas directas a sus preguntas. (No haré el álgebra ya que, si puede realizar los cálculos para la ecuación de Dirac en sí, entonces la transformación del Lagrangiano debería ser sencilla).

"Supongamos que operamos C en el Lagrangiano de Dirac, ¿qué deberíamos obtener?" La relación corregida es:

C L D i r a C ( X m ) C 1 = L D i r a C ( X m )

(Dicho sea de paso, su relación resulta ser correcta ya que el lagrangiano (densidad) debe ser un operador escalar hermitiano, por lo que L = L = L . EDITAR: Gracias a Omkar por señalar que esto está mal. L L . )

En la misma nota, uno puede preguntar ¿qué ecuación debe satisfacer ψC? En caso de que satisfaga la ecuación de Dirac conjugada como

> ( i γ m m + metro ) ψ C = 0 ?

Si usa la métrica "Costa oeste" ( + 1 , 1 , 1 , 1 ) entonces la ecuacion que ψ C debe satisfacer es el de arriba con metro metro . Es decir, la ecuación libre de Dirac es la misma para ψ y ψ C . Esto se debe a que las masas de la partícula y la antipartícula son idénticas, como lo adivinó por primera vez Dirac. (Si está utilizando la métrica del signo opuesto, entonces su ecuación es correcta).

Gracias por su respuesta. Estoy de acuerdo con tu argumento con respecto al Lagrangiano. E incluso su argumento con respecto a la forma de la ecuación suena correcto y algo que deberíamos esperar dado que el Lagrangiano es el mismo. Sin embargo, hice un cálculo y encontré lo contrario. Pondré el cálculo en la pregunta, tal vez esté cometiendo algún error.
otro punto es que aunque L es hermético, creo que no es real, como el término ψ ¯ ψ no es real. Por lo tanto, no puedes decir L = L
Veo un error en lo anterior. Deberías: γ 0 γ m m = ( γ 0 t γ i i ) γ 0 . Aunque yo no lo haría de esta manera. Obtengo el resultado correcto comenzando con la ecuación de Dirac para ψ , tomando el complejo conjugado y demostrando que C γ 2 (en lo que a veces se llama la representación de Dirac del γ matrices). Y tienes razón en que L L . Lo siento por eso.
Lo siento de nuevo -- C debiera ser γ 2 γ 0 . (Estoy usando las convenciones de Bjorken & Drell).
Si tienes razón mis anotaciones son malas, estoy usando un libro muy malo :\ . Por cierto γ 0 γ m m = γ 0 γ 0 t γ 0 γ i i = γ 0 γ 0 t + γ i γ 0 i = ( γ 0 t + γ i i ) γ 0
Omkar, este es un error común que cometiste en la línea anterior para γ m m . Considerar a m b m = a 0 b 0 + a i b i -- no hay signo menos ya que aún no has bajado el índice. La relación correcta es: γ m m = γ 0 0 i = 1 3 γ i i .

Depende de la definición del operador de conjugación de carga. tu definición

ψ C = C γ 0 T ψ .
cambia el signo del término de masa en la ecuación de Dirac (o, de manera equivalente, el signo del término cinético si multiplicamos la ecuación por un signo menos). Si añadimos un extra γ 5 (o i γ 5 , lo que equivale a una transformación de fase global.) a su definición de conjugación de carga,
ψ C = γ 5 C γ 0 T ψ ,
¡voilá! la conjugación de carga lo devolvería a la ecuación original de Dirac en virtud de
γ 5 γ m = γ m γ 5 .